OB unit 3 exam

¡Supera tus tareas y exámenes ahora con Quizwiz!

Which assessment is least likely to be associated with a breech presentation? a. Meconium-stained amniotic fluid b. Fetal heart tones heard at or above the maternal umbilicus c. Preterm labor and birth d. Postterm gestation

Postterm gestation ANS: D Postterm gestation is not likely to occur with a breech presentation. The presence of meconium in a breech presentation may be a result of pressure on the fetal wall as it traverses the birth canal. Fetal heart tones heard at the level of the umbilical level of the mother are a typical finding in a breech presentation because the fetal back would be located in the upper abdominal area. Breech presentations often occur in preterm births.

PPH may be sudden and result in rapid blood loss. The nurse must be alert to the symptoms of hemorrhage and hypovolemic shock and be prepared to act quickly to minimize blood loss. Astute assessment of the clients circulatory status can be performed with noninvasive monitoring. Match the type of noninvasive assessment that the nurse would perform with the appropriate clinical manifestation or body system. 1. Pulse oximetry 2. Heart sounds 3. Arterial pulses 4. Skin color, temperature, and turgor 5. Presence or absence of anxiety a. Palpation b. Auscultation c. Inspection d. Observation e. Measurement

1. Pulse oximetry E 2. Heart sounds B 3. Arterial pulses A 4. Skin color, temperature, and turgor C 5. Presence or absence of anxiety D

The nurse is preparing to teach an antepartum client with GDM the correct method of administering an intermediate-acting insulin, such as neutral protamine Hagedorn (NPH), with a short-acting insulin (regular). In the correct order from 1 through 6, match the step number with the action needed to teach the client self-administration of this combination of insulin. 1. Step 1 2. Step 2 3. Step 3 4. Step 4 5. Step 5 6. Step 6 a. Without adding air, withdraw the correct dose of NPH insulin. b. Gently rotate the insulin to mix it, and wipe the stopper. c. Inject air equal to the dose of NPH insulin into the vial, and remove the syringe. d. Inject air equal to the dose of regular insulin into the vial, and withdraw the medication. e. Check the insulin bottles for the expiration date. f. Wash hands.

1. Step 1 F 2. Step 2 E 3. Step 3 B 4. Step 4 C 5. Step 5 D 6. Step 6 A

Lacerations of the cervix, vagina, or perineum are also causes of PPH. Which factors influence the causes and incidence of obstetric lacerations of the lower genital tract? (Select all that apply.) a. Operative and precipitate births b. Adherent retained placenta c. Abnormal presentation of the fetus d. Congenital abnormalities of the maternal soft tissue e. Previous scarring from infection

A, C, D, E Abnormal adherence of the placenta occurs for unknown reasons. Attempts to remove the placenta in the usual manner can be unsuccessful, and lacerations or a perforation of the uterine wall may result. However, attempts to remove the placenta do not influence lower genital tract lacerations. Lacerations of the perineum are the most common of all lower genital tract injuries and often occur with both precipitate and operative births and are classified as first-, second-, third-, and fourth-degree lacerations. An abnormal presentation or position of the fetus, the relative size of the presenting part, and the birth canal may contribute to lacerations of the lower genital tract. Congenital abnormalities, previous scarring from infection or injury, and a contracted pelvis may also influence injury to the lower genital tract, followed by hemorrhage.

Achieving and maintaining euglycemia are the primary goals of medical therapy for the pregnant woman with diabetes. These goals are achieved through a combination of diet, insulin, exercise, and blood glucose monitoring. The target blood glucose levels 1 hour after a meal should be _____________.

ANS: 110 to 129 mg/dl Target levels of blood glucose during pregnancy are lower than nonpregnant values. Accepted fasting levels are between 60 and 99 mg/dl, and 1-hour postmeal levels should be between 110 to 129 mg/dl. Two-hour postmeal levels should be 120 mg/dl or less.

When is a prophylactic cerclage for an incompetent cervix usually placed (in weeks of gestation)? a. 12 to 14 b. 6 to 8 c. 23 to 24 d. After 24

ANS: A A prophylactic cerclage is usually placed at 12 to 14 weeks of gestation. The cerclage is electively removed when the woman reaches 37 weeks of gestation or when her labor begins. Six to 8 weeks of gestation is too early to place the cerclage. Cerclage placement is offered if the cervical length falls to less than 20 to 25 mm before 23 to 24 weeks. Although no consensus has been reached, 24 weeks is used as the upper gestational age limit for cerclage placement.

A client at 39 weeks of gestation has been admitted for an external version. Which intervention would the nurse anticipate the provider to order? a. Tocolytic drug b. Contraction stress test (CST) c. Local anesthetic d. Foley catheter

ANS: A A tocolytic drug will relax the uterus before and during the version, thus making manipulation easier. CST is used to determine the fetal response to stress. A local anesthetic is not used with external version. Although the bladder should be emptied, catheterization is not necessary.

What is the correct definition of a spontaneous termination of a pregnancy (abortion)? a. Pregnancy is less than 20 weeks. b. Fetus weighs less than 1000 g. c. Products of conception are passed intact. d. No evidence exists of intrauterine infection.

ANS: A An abortion is the termination of pregnancy before the age of viability (20 weeks). The weight of the fetus is not considered because some older fetuses may have a low birth weight. A spontaneous abortion may be complete or incomplete and may be caused by many problems, one being intrauterine infection.

What are the complications and risks associated with cesarean births? (Select all that apply.) a. Pulmonary edema b. Wound dehiscence c. Hemorrhage d. Urinary tract infections e. Fetal injuries

ANS: A, B, C, D, E Pulmonary edema, wound dehiscence, hemorrhage, urinary tract infections, and fetal injuries are possible complications and risks associated with cesarean births.

Women who are obese are at risk for several complications during pregnancy and birth. Which of these would the nurse anticipate with an obese client? (Select all that apply.) a. Thromboembolism b. Cesarean birth c. Wound infection d. Breech presentation e. Hypertension

ANS: A, B, C, E A breech presentation is not a complication of pregnancy or birth for the client who is obese. Venous thromboembolism is a known risk for obese women. Therefore, the use of thromboembolism-deterrent (TED) hose and sequential compression devices may help decrease the chance for clot formation. Women should also be encouraged to ambulate as soon as possible. In addition to having an increased risk for complications with a cesarean birth, in general, obese women are also more likely to require an emergency cesarean birth. Many obese women have a pannus (i.e., large roll of abdominal fat) that overlies a lower transverse incision made just above the pubic area. The pannus causes the area to remain moist, which encourages the development of infection. Obese women are more likely to begin pregnancy with comorbidities such as hypertension and type 2 diabetes.

Which medications are used to manage PPH? (Select all that apply.) a. Oxytocin b. Methergine c. Terbutaline d. Hemabate e. Magnesium sulfate

ANS: A, B, D Oxytocin, Methergine, and Hemabate are medications used to manage PPH. Terbutaline and magnesium sulfate are tocolytic medications that are used to relax the uterus, which would cause or worsen PPH.

Screening questions for alcohol and drug abuse should be included in the overall assessment during the first prenatal visit for all women. The 4 Ps Plus is a screening tool specifically designed to identify the need for a more in-depth assessment. Which are the correct components of the 4 Ps Plus? (Select all that apply.) a. Parents b. Partner c. Present d. Past e. Pregnancy

ANS: A, B, D, E The nurse who is screening the client using the 4 Ps Plus would use the following format: Parents: Did either of your parents have a problem with alcohol or drugs? Partner: Does your partner have a problem with alcohol or drugs? Past: Have you ever had any beer, wine, or liquor? Pregnancy: In the month before you knew you were pregnant, how many cigarettes did you smoke? How much beer, wine, or liquor did you drink? Present: Is not a component of the 4 Ps Plus.

A serious but uncommon complication of undiagnosed or partially treated hyperthyroidism is a thyroid storm, which may occur in response to stress such as infection, birth, or surgery. What are the signs and symptoms of this emergency disorder? (Select all that apply.) a. Fever b. Hypothermia c. Restlessness d. Bradycardia e. Hypertension

ANS: A, C Fever, restlessness, tachycardia, vomiting, hypotension, and stupor are symptoms of a thyroid storm. Fever, not hypothermia; tachycardia, not bradycardia; and hypotension, not hypertension, are symptoms of thyroid storm.

Which adverse prenatal outcomes are associated with the HELLP syndrome? (Select all that apply.) a. Placental abruption b. Placenta previa c. Renal failure d. Cirrhosis e. Maternal and fetal death

ANS: A, C, E The HELLP syndrome is associated with an increased risk for adverse perinatal outcomes, including placental abruption, acute renal failure, subcapsular hepatic hematoma, hepatic rupture, recurrent preeclampsia, preterm birth, and fetal and maternal death. The HELLP syndrome is associated with an increased risk for placental abruption, not placenta previa. It is also associated with an increased risk for hepatic hematoma, not cirrhosis.

What is a maternal indication for the use of vacuum-assisted birth? a. Wide pelvic outlet b. Maternal exhaustion c. History of rapid deliveries d. Failure to progress past station 0

ANS: B A mother who is exhausted may be unable to assist with the expulsion of the fetus. The client with a wide pelvic outlet will likely not require vacuum extraction. With a rapid delivery, vacuum extraction is not necessary. A station of 0 is too high for a vacuum-assisted birth.

The client being cared for has severe preeclampsia and is receiving a magnesium sulfate infusion. Which new finding would give the nurse cause for concern? a. Sleepy, sedated affect b. Respiratory rate of 10 breaths per minute c. DTRs of 2 d. Absent ankle clonus

ANS: B A respiratory rate of 10 breaths per minute indicates the client is experiencing respiratory depression from magnesium toxicity. Because magnesium sulfate is a CNS depressant, the client will most likely become sedated when the infusion is initiated. DTRs of 2 and absent ankle clonus are normal findings.

A client with a history of bipolar disorder is called by the postpartum support nurse for follow-up. Which symptoms would reassure the nurse that the client is not experiencing a manic episode? a. Psychomotor agitation and lack of sleep b. Increased appetite and lack of interest in activities c. Hyperactivity and distractibility d. Pressured speech and grandiosity

ANS: B An increased appetite and a lack of interest would reassure the nurse that the client is not experiencing an episode of mania. Clinical manifestations of a manic episode include at least three of the following: grandiosity, decreased need for sleep, pressured speech, flight of ideas, distractibility, psychomotor agitation, and excessive involvement in pleasurable activities. The pregnant woman exhibiting symptoms of a manic episode will likely have a decreased interest in eating and an increased level of interest in pleasurable activities without regard for negative consequences. Psychomotor agitation and a lack of sleep, hyperactivity and distractibility, and pressured speech and grandiosity are all clinical manifestations of a manic episode.

A number of methods can be used for inducing labor. Which cervical ripening method falls under the category of mechanical or physical? a. Prostaglandins are used to soften and thin the cervix. b. Labor can sometimes be induced with balloon catheters or laminaria tents. c. Oxytocin is less expensive and more effective than prostaglandins but creates greater health risks. d. Amniotomy can be used to make the cervix more favorable for labor.

ANS: B Balloon catheters or laminaria tents are mechanical means of ripening the cervix. Ripening the cervix, making it softer and thinner, increases the success rate of induced labor. Prostaglandin E1 is less expensive and more effective than oxytocin but carries a greater risk. Amniotomy is the artificial rupture of membranes, which is used to induce labor only when the cervix is already ripe.

In contrast to placenta previa, what is the most prevalent clinical manifestation of abruptio placentae? a. Bleeding b. Intense abdominal pain c. Uterine activity d. Cramping

ANS: B Pain is absent with placenta previa and may be agonizing with abruptio placentae. Bleeding may be present in varying degrees for both placental conditions. Uterine activity and cramping may be present with both placental conditions.

What is the primary nursing responsibility when caring for a client who is experiencing an obstetric hemorrhage associated with uterine atony? a. Establishing venous access b. Performing fundal massage c. Preparing the woman for surgical intervention d. Catheterizing the bladder

ANS: B The initial management of excessive postpartum bleeding is a firm massage of the uterine fundus. Although establishing venous access may be a necessary intervention, fundal massage is the initial intervention. The woman may need surgical intervention to treat her postpartum hemorrhage, but the initial nursing intervention is to assess the uterus. After uterine massage, the nurse may want to catheterize the client to eliminate any bladder distention that may be preventing the uterus from properly contracting.

In terms of the incidence and classification of diabetes, which information should the nurse keep in mind when evaluating clients during their ongoing prenatal appointments? a. Type 1 diabetes is most common. b. Type 2 diabetes often goes undiagnosed. c. GDM means that the woman will receive insulin treatment until 6 weeks after birth. d. Type 1 diabetes may become type 2 during pregnancy.

ANS: B Type 2 diabetes often goes undiagnosed because hyperglycemia gradually develops and is often not severe. Type 2, sometimes called adult-onset diabetes, is the most common type of diabetes. GDM refers to any degree of glucose intolerance first recognized during pregnancy; insulin may or may not be needed. People do not go back and forth between type 1 and type 2 diabetes.

A laboring woman with no known risk factors suddenly experiences spontaneous ROM. The fluid consists of bright red blood. Her contractions are consistent with her current stage of labor. No change in uterine resting tone has occurred. The fetal heart rate (FHR) begins to decline rapidly after the ROM. The nurse should suspect the possibility of what condition? a. Placenta previa b. Vasa previa c. Severe abruptio placentae d. Disseminated intravascular coagulation (DIC)

ANS: B Vasa previa is the result of a velamentous insertion of the umbilical cord. The umbilical vessels are not surrounded by Wharton jelly and have no supportive tissue. The umbilical blood vessels thus are at risk for laceration at any time, but laceration occurs most frequently during ROM. The sudden appearance of bright red blood at the time of ROM and a sudden change in the FHR without other known risk factors should immediately alert the nurse to the possibility of vasa previa. The presence of placenta previa most likely would be ascertained before labor and is considered a risk factor for this pregnancy. In addition, if the woman had a placenta previa, it is unlikely that she would be allowed to pursue labor and a vaginal birth. With the presence of severe abruptio placentae, the uterine tonicity typically is tetanus (i.e., a boardlike uterus). DIC is a pathologic form of diffuse clotting that consumes large amounts of clotting factors, causing widespread external bleeding, internal bleeding, or both. DIC is always a secondary diagnosis, often associated with obstetric risk factors such as the hemolysis, elevated liver enzyme levels, and low platelet levels (HELLP) syndrome. This woman did not have any prior risk factors.

Indications for a primary cesarean birth are often nonrecurring. Therefore, a woman who has had a cesarean birth with a low transverse scar may be a candidate for vaginal birth after cesarean (VBAC). Which clients would beless likely to have a successful VBAC? (Select all that apply.) a. Lengthy interpregnancy interval b. African-American race c. Delivery at a rural hospital d. Estimated fetal weight <4000 g e. Maternal obesity (BMI >30)

ANS: B, C, E Indications for a low success rate for a VBAC delivery include a short interpregnancy interval, non-Caucasian race, gestational age longer than 40 weeks, maternal obesity, preeclampsia, fetal weight greater than 4000 g, and delivery at a rural or private hospital.

Which is the initial treatment for the client with vWD who experiences a PPH? a. Cryoprecipitate b. Factor VIII and von Willebrand factor (vWf) c. Desmopressin d. Hemabate

ANS: C Desmopressin is the primary treatment of choice for vWD and can be administered orally, nasally, and intravenously. This medication promotes the release of factor VIII and vWf from storage. Cryoprecipitate may be used; however, because of the risk of possible donor viruses, other modalities are considered safer. Treatment with plasma products such as factor VIII and vWf is an acceptable option for this client. Because of the repeated exposure to donor blood products and possible viruses, this modality is not the initial treatment of choice. Although the administration of the prostaglandin, Hemabate, is known to promote contraction of the uterus during PPH, it is not effective for the client who has a bleeding disorder.

A pregnant woman who abuses cocaine admits to exchanging sex to finance her drug habit. This behavior places the client at the greatest risk for what? a. Depression of the CNS b. Hypotension and vasodilation c. Sexually transmitted infections (STIs) d. Postmature birth

ANS: C Exchanging sex acts for drugs places the woman at increased risk for STIs because of multiple partners and the lack of protection. Cocaine is a CNS stimulant that causes hypertension and vasoconstriction. Premature delivery of the infant is one of the more common problems associated with cocaine use during pregnancy.

Which is the most accurate description of PPD without psychotic features? a. Postpartum baby blues requiring the woman to visit with a counselor or psychologist b. Condition that is more common among older Caucasian women because they have higher expectations c. Distinguishable by pervasive sadness along with mood swings d. Condition that disappears without outside help

ANS: C PPD is characterized by an intense pervasive sadness along with labile mood swings and is more persistent than postpartum baby blues. PPD, even without psychotic features, is more serious and persistent than postpartum baby blues. PPD is more common among younger mothers and African-American mothers. Most women need professional help to get through PPD, including pharmacologic intervention.

Reports have linked third trimester use of selective serotonin uptake inhibitors (SSRIs) with a constellation of neonatal signs. The nurse is about to perform an assessment on the infant of a mother with a history of a mood disorder. Which signs and symptoms in the neonate may be the result of maternal SSRI use? (Select all that apply.) a. Hypotonia b. Hyperglycemia c. Shivering d. Fever e. Irritability

ANS: C, D, E Neonatal signs of maternal SSRI use include continuous crying, irritability, jitteriness, shivering, fever, hypertonia, respiratory distress, feeding difficulty, hypoglycemia, and seizures. The onset of signs and symptoms ranges from several hours to several days after birth, but the signs generally resolve within 2 weeks.

Hypothyroidism occurs in 2 to 3 pregnancies per 1000. Because severe hypothyroidism is associated with infertility and miscarriage, it is not often seen in pregnancy. Regardless of this fact, the nurse should be aware of the characteristic symptoms of hypothyroidism. Which do they include? (Select all that apply.) a. Hot flashes b. Weight loss c. Lethargy d. Decrease in exercise capacity e. Cold intolerance

ANS: C, D, E Symptoms include weight gain, lethargy, decrease in exercise capacity, and intolerance to cold. Other presentations might include constipation, hoarseness, hair loss, and dry skin. Thyroid supplements are used to treat hyperthyroidism in pregnancy.

A woman with gestational diabetes has had little or no experience reading and interpreting glucose levels. The client shows the nurse her readings for the past few days. Which reading signals the nurse that the client may require an adjustment of insulin or carbohydrates? a. 75 mg/dl before lunch. This is low; better eat now. b. 115 mg/dl 1 hour after lunch. This is a little high; maybe eat a little less next time. c. 115 mg/dl 2 hours after lunch. This is too high; it is time for insulin. d. 50 mg/dl just after waking up from a nap. This is too low; maybe eat a snack before going to sleep.

ANS: D 50 mg/dl after waking from a nap is too low. During hours of sleep, glucose levels should not be less than 60 mg/dl. Snacks before sleeping can be helpful. The premeal acceptable range is 60 to 99 mg/dl. The readings 1 hour after a meal should be less than 129 mg/dl. Two hours after eating, the readings should be less than 120 mg/dl.

The nurse who elects to practice in the area of womens health must have a thorough understanding of miscarriage. Which statement regarding this condition is most accurate? a. A miscarriage is a natural pregnancy loss before labor begins. b. It occurs in fewer than 5% of all clinically recognized pregnancies. c. Careless maternal behavior, such as poor nutrition or excessive exercise, can be a factor in causing a miscarriage. d. If a miscarriage occurs before the 12th week of pregnancy, then it may be observed only as moderate discomfort and blood loss.

ANS: D Before the sixth week, the only evidence might be a heavy menstrual flow. After the 12th week, more severe pain, similar to that of labor, is likely. Miscarriage is a natural pregnancy loss, but it occurs, by definition, before 20 weeks of gestation, before the fetus is viable. Miscarriages occur in approximately 10% to 15% of all clinically recognized pregnancies. Miscarriages can be caused by a number of disorders or illnesses outside the mothers control or knowledge.

Which finding on a prenatal visit at 10 weeks of gestation might suggest a hydatidiform mole? a. Complaint of frequent mild nausea b. Blood pressure of 120/80 mm Hg c. Fundal height measurement of 18 cm d. History of bright red spotting for 1 day, weeks ago

Which finding on a prenatal visit at 10 weeks of gestation might suggest a hydatidiform mole? a. Complaint of frequent mild nausea b. Blood pressure of 120/80 mm Hg c. Fundal height measurement of 18 cm d. History of bright red spotting for 1 day, weeks ago

A woman in preterm labor at 30 weeks of gestation receives two 12-mg intramuscular (IM) doses of betamethasone. What is the purpose of this pharmacologic intervention? a. To stimulate fetal surfactant production b. To reduce maternal and fetal tachycardia associated with ritodrine administration c. To suppress uterine contractions d. To maintain adequate maternal respiratory effort and ventilation during magnesium sulfate therapy

ANS: A Antenatal glucocorticoids administered as IM injections to the mother accelerate fetal lung maturity. Propranolol (Inderal) is given to reduce the effects of ritodrine administration. Betamethasone has no effect on uterine contractions. Calcium gluconate is given to reverse the respiratory depressive effects of magnesium sulfate therapy.

What is the highest priority nursing intervention when admitting a pregnant woman who has experienced a bleeding episode in late pregnancy? a. Assessing FHR and maternal vital signs b. Performing a venipuncture for hemoglobin and hematocrit levels c. Placing clean disposable pads to collect any drainage d. Monitoring uterine contractions

ANS: A Assessment of the FHR and maternal vital signs will assist the nurse in determining the degree of the blood loss and its effect on the mother and fetus. The most important assessment is to check the well-being of both the mother and the fetus. The blood levels can be obtained later. Assessing future bleeding is important; however, the top priority remains mother/fetal well-being. Monitoring uterine contractions is important but not a top priority.

Which description most accurately describes the augmentation of labor? a. Is part of the active management of labor that is instituted when the labor process is unsatisfactory b. Relies on more invasive methods when oxytocin and amniotomy have failed c. Is a modern management term to cover up the negative connotations of forceps-assisted birth d. Uses vacuum cups

ANS: A Augmentation is part of the active management of labor that stimulates uterine contractions after labor has started but is not progressing satisfactorily. Augmentation uses amniotomy and oxytocin infusion, as well as some more gentle, noninvasive methods. Forceps-assisted births are less common than in the past and not considered a method of augmentation. A vacuum-assisted delivery occurs during childbirth if the mother is too exhausted to push. Vacuum extraction is not considered an augmentation methodology.

The client is being induced in response to worsening preeclampsia. She is also receiving magnesium sulfate. It appears that her labor has not become active, despite several hours of oxytocin administration. She asks the nurse, Why is this taking so long? What is the nurses most appropriate response? a. The magnesium is relaxing your uterus and competing with the oxytocin. It may increase the duration of your labor. b. I dont know why it is taking so long. c. The length of labor varies for different women. d. Your baby is just being stubborn.

ANS: A Because magnesium sulfate is a tocolytic agent, its use may increase the duration of labor. The amount of oxytocin needed to stimulate labor may be more than that needed for the woman who is not receiving magnesium sulfate. The nurse should explain to the client the effects of magnesium sulfate on the duration of labor. Although the length of labor varies for different women, the most likely reason this womans labor is protracted is the tocolytic effects of magnesium sulfate. The behavior of the fetus has no bearing on the length of labor.

Which intervention is most important when planning care for a client with severe gestational hypertension? a. Induction of labor is likely, as near term as possible. b. If at home, the woman should be confined to her bed, even with mild gestational hypertension. c. Special diet low in protein and salt should be initiated. d. Vaginal birth is still an option, even in severe cases.

ANS: A By 34 weeks of gestation, the risk of continuing the pregnancy may be considered greater than the risks of a preterm birth. Strict bed rest is controversial for mild cases; some women in the hospital are even allowed to move around. Diet and fluid recommendations are essentially the same as for healthy pregnant women, although some authorities have suggested a diet high in protein. Women with severe gestational hypertension should expect a cesarean delivery.

Which order should the nurse expect for a client admitted with a threatened abortion? a. Bed rest b. Administration of ritodrine IV c. Nothing by mouth (nil per os [NPO]) d. Narcotic analgesia every 3 hours, as needed

ANS: A Decreasing the womans activity level may alleviate the bleeding and allow the pregnancy to continue. Ritodrine is not the first drug of choice for tocolytic medications. Having the woman placed on NPO is unnecessary. At times, dehydration may produce contractions; therefore, hydration is important. Narcotic analgesia will not decrease the contractions and may mask the severity of the contractions.

A woman with severe preeclampsia has been receiving magnesium sulfate by intravenous infusion for 8 hours. The nurse assesses the client and documents the following findings: temperature of 37.1 C, pulse rate of 96 beats per minute, respiratory rate of 24 breaths per minute, BP of 155/112 mm Hg, 3+ DTRs, and no ankle clonus. The nurse calls the provider with an update. The nurse should anticipate an order for which medication? a. Hydralazine b. Magnesium sulfate bolus c. Diazepam d. Calcium gluconate

ANS: A Hydralazine is an antihypertensive medication commonly used to treat hypertension in severe preeclampsia. Typically, it is administered for a systolic BP higher than 160 mm Hg or a diastolic BP higher than 110 mm Hg. An additional bolus of magnesium sulfate may be ordered for increasing signs of CNS irritability related to severe preeclampsia (e.g., clonus) or if eclampsia develops. Diazepam is sometimes used to stop or shorten eclamptic seizures. Calcium gluconate is used as the antidote for magnesium sulfate toxicity. The client is not currently displaying any signs or symptoms of magnesium toxicity.

The exact cause of preterm labor is unknown but believed to be multifactorial. Infection is thought to be a major factor in many preterm labors. Which type of infection has not been linked to preterm birth? a. Viral b. Periodontal c. Cervical d. Urinary tract

ANS: A Infections that increase the risk of preterm labor and birth are bacterial and include cervical, urinary tract, periodontal, and other bacterial infections. Therefore, early, continual, and comprehensive participation by the client in her prenatal care is important. Recent evidence has shown a link between periodontal infections and preterm labor. Researchers recommend regular dental care before and during pregnancy, oral assessment as a routine part of prenatal care, and scrupulous oral hygiene to prevent periodontal infections.

Which condition is considered a medical emergency that requires immediate treatment? a. Inversion of the uterus b. Hypotonic uterus c. ITP d. Uterine atony

ANS: A Inversion of the uterus is likely to lead to hypovolemic shock and therefore is considered a medical emergency. Although hypotonic uterus, ITP, and uterine atony are serious conditions, they are not necessarily medical emergencies that require immediate treatment.

What is the most common reason for late postpartum hemorrhage (PPH)? a. Subinvolution of the uterus b. Defective vascularity of the decidua c. Cervical lacerations d. Coagulation disorders

ANS: A Late PPH may be the result of subinvolution of the uterus. Recognized causes of subinvolution include retained placental fragments and pelvic infection. Although defective vascularity, cervical lacerations, and coagulation disorders of the decidua may also cause PPH, late PPH typically results from subinvolution of the uterus, pelvic infection, or retained placental fragments.

With one exception, the safest pregnancy is one during which the woman is drug and alcohol free. What is the optimal treatment for women addicted to opioids? a. Methadone maintenance treatment (MMT) b. Detoxification c. Smoking cessation d. 4 Ps Plus

ANS: A MMT is currently considered the standard of care for pregnant women who are dependent on heroin or other narcotics. Buprenorphine is another medication approved for the treatment of opioid addiction that is increasingly being used during pregnancy. Opioid replacement therapy has been shown to decrease opioid and other drug use, reduce criminal activity, improve individual functioning, and decrease the rates of infections such as hepatitis B and C, human immunodeficiency virus (HIV), and other STIs. Detoxification is the treatment used for alcohol addiction. Pregnant women requiring withdrawal from alcohol should be admitted for inpatient management. Women are more likely to stop smoking during pregnancy than at any other time in their lives. A smoking cessation program can assist in achieving this goal. The 4 Ps Plus is a screening tool specifically designed to identify pregnant women who need in-depth assessment related to substance abuse.

Which neonatal complications are associated with hypertension in the mother? a. Intrauterine growth restriction (IUGR) and prematurity b. Seizures and cerebral hemorrhage c. Hepatic or renal dysfunction d. Placental abruption and DIC

ANS: A Neonatal complications are related to placental insufficiency and include IUGR, prematurity, and necrotizing enterocolitis. Seizures and cerebral hemorrhage are maternal complications. Hepatic and renal dysfunction are maternal complications of hypertensive disorders in pregnancy. Placental abruption and DIC are conditions related to maternal morbidity and mortality.

Screening at 24 weeks of gestation reveals that a pregnant woman has gestational diabetes mellitus (GDM). In planning her care, the nurse and the client mutually agree that an expected outcome is to prevent injury to the fetus as a result of GDM. This fetus is at the greatest risk for which condition? a. Macrosomia b. Congenital anomalies of the central nervous system c. Preterm birth d. Low birth weight

ANS: A Poor glycemic control later in pregnancy increases the rate of fetal macrosomia. Poor glycemic control during the preconception time frame and into the early weeks of the pregnancy is associated with congenital anomalies. Preterm labor or birth is more likely to occur with severe diabetes and is the greatest risk in women with pregestational diabetes. Increased weight, or macrosomia, is the greatest risk factor for this fetus.

What is the most common medical complication of pregnancy? a. Hypertension b. Hyperemesis gravidarum c. Hemorrhagic complications d. Infections

ANS: A Preeclampsia and eclampsia are two noted deadly forms of hypertension. A large percentage of pregnant women will have nausea and vomiting, but a relatively few will have the severe form called hyperemesis gravidarum. Hemorrhagic complications are the second most common medical complication of pregnancy; hypertension is the most common. Infection is a risk factor for preeclampsia.

According to research, which risk factor for PPD is likely to have the greatest effect on the client postpartum? a. Prenatal depression b. Single-mother status c. Low socioeconomic status d. Unplanned or unwanted pregnancy

ANS: A Prenatal depression has been found to be a major risk factor for PPD. Single-mother status and low socioeconomic status are both small-relationship predictors for PPD. Although an unwanted pregnancy may contribute to the risk for PPD, it does not pose as great an effect as prenatal depression.

In caring for the woman with DIC, which order should the nurse anticipate? a. Administration of blood b. Preparation of the client for invasive hemodynamic monitoring c. Restriction of intravascular fluids d. Administration of steroids

ANS: A Primary medical management in all cases of DIC involves a correction of the underlying cause, volume replacement, blood component therapy, optimization of oxygenation and perfusion status, and continued reassessment of laboratory parameters. Central monitoring would not be initially ordered in a client with DIC because it could contribute to more areas of bleeding. Management of DIC would include volume replacement, not volume restriction. Steroids are not indicated for the management of DIC.

Which statement concerning the complication of maternal diabetes is the most accurate? a. Diabetic ketoacidosis (DKA) can lead to fetal death at any time during pregnancy. b. Hydramnios occurs approximately twice as often in diabetic pregnancies than in nondiabetic pregnancies. c. Infections occur about as often and are considered about as serious in both diabetic and nondiabetic pregnancies. d. Even mild-to-moderate hypoglycemic episodes can have significant effects on fetal well-being.

ANS: A Prompt treatment of DKA is necessary to save the fetus and the mother. Hydramnios occurs 10 times more often in diabetic pregnancies. Infections are more common and more serious in pregnant women with diabetes. Mild-to-moderate hypoglycemic episodes do not appear to have significant effects on fetal well-being.

Despite warnings, prenatal exposure to alcohol continues to far exceed exposure to illicit drugs. Which condition is rarely associated with fetal alcohol syndrome (FAS)? a. Respiratory conditions b. Intellectual impairment c. Neural development disorder d. Alcohol-related birth defects (ARBDs)

ANS: A Respiratory difficulties are not attributed to exposure to alcohol in utero. Other abnormalities related to FAS include mental retardation, neurodevelopment disorders, and ARBDs.

What nursing diagnosis is the most appropriate for a woman experiencing severe preeclampsia? a. Risk for injury to mother and fetus, related to central nervous system (CNS) irritability b. Risk for altered gas exchange c. Risk for deficient fluid volume, related to increased sodium retention secondary to the administration of magnesium sulfate d. Risk for increased cardiac output, related to the use of antihypertensive drugs

ANS: A Risk for injury is the most appropriate nursing diagnosis for this client scenario. Gas exchange is more likely to become impaired, attributable to pulmonary edema. A risk for excess, not deficient, fluid volume, related to increased sodium retention, is increased, and a risk for decreased, not increased, cardiac output, related to the use of antihypertensive drugs, also is increased.

With regard to hemorrhagic complications that may occur during pregnancy, what information is most accurate? a. An incompetent cervix is usually not diagnosed until the woman has lost one or two pregnancies. b. Incidences of ectopic pregnancy are declining as a result of improved diagnostic techniques. c. One ectopic pregnancy does not affect a womans fertility or her likelihood of having a normal pregnancy the next time. d. Gestational trophoblastic neoplasia (GTN) is one of the persistently incurable gynecologic malignancies.

ANS: A Short labors and recurring losses of pregnancy at progressively earlier gestational ages are characteristics of reduced cervical competence. Because diagnostic technology is improving, more ectopic pregnancies are being diagnosed. One ectopic pregnancy places the woman at increased risk for another one. Ectopic pregnancy is a leading cause of infertility. Once invariably fatal, GTN now is the most curable gynecologic malignancy.

In caring for an immediate postpartum client, the nurse notes petechiae and oozing from her intravenous (IV) site. The client would be closely monitored for which clotting disorder? a. DIC b. Amniotic fluid embolism (AFE) c. Hemorrhage d. HELLP syndrome

ANS: A The diagnosis of DIC is made according to clinical findings and laboratory markers. A physical examination reveals unusual bleeding. Petechiae may appear around a blood pressure cuff on the womans arm. Excessive bleeding may occur from the site of slight trauma such as venipuncture sites. These symptoms are not associated with AFE, nor is AFE a bleeding disorder. Hemorrhage occurs for a variety of reasons in the postpartum client. These symptoms are associated with DIC. Hemorrhage would be a finding associated with DIC and is not a clotting disorder in and of itself. HELLP syndrome is not a clotting disorder, but it may contribute to the clotting disorder DIC.

Which nursing intervention should be immediately performed after the forceps-assisted birth of an infant? a. Assessing the infant for signs of trauma b. Administering prophylactic antibiotic agents to the infant c. Applying a cold pack to the infants scalp d. Measuring the circumference of the infants head

ANS: A The infant should be assessed for bruising or abrasions at the site of application, facial palsy, and subdural hematoma. Prophylactic antibiotics are not necessary with a forceps delivery. A cold pack would place the infant at risk for cold stress and is contraindicated. Measuring the circumference of the head is part of the initial nursing assessment.

Which major neonatal complication is carefully monitored after the birth of the infant of a diabetic mother? a. Hypoglycemia b. Hypercalcemia c. Hypobilirubinemia d. Hypoinsulinemia

ANS: A The neonate is at highest risk for hypoglycemia because fetal insulin production is accelerated during pregnancy to metabolize excessive glucose from the mother. At birth, the maternal glucose supply stops and the neonatal insulin exceeds the available glucose, thus leading to hypoglycemia. Hypocalcemia is associated with preterm birth, birth trauma, and asphyxia, all common problems of the infant of a diabetic mother. Excess erythrocytes are broken down after birth, and large amounts of bilirubin are released into the neonates circulation, with resulting hyperbilirubinemia. Because fetal insulin production is accelerated during pregnancy, hyperinsulinemia develops in the neonate.

A pregnant womans amniotic membranes have ruptured. A prolapsed umbilical cord is suspected. What intervention would be the nurses highest priority? a. Placing the woman in the knee-chest position b. Covering the cord in sterile gauze soaked in saline c. Preparing the woman for a cesarean birth d. Starting oxygen by face mask

ANS: A The woman is assisted into a modified Sims position, Trendelenburg position, or the knee-chest position in which gravity keeps the pressure of the presenting part off the cord. Although covering the cord in sterile gauze soaked saline, preparing the woman for a cesarean, and starting oxygen by face mark are appropriate nursing interventions in the event of a prolapsed cord, the intervention of top priority would be positioning the mother to relieve cord compression.

A perinatal nurse is caring for a woman in the immediate postbirth period. Assessment reveals that the client is experiencing profuse bleeding. What is the most likely cause for this bleeding? a. Uterine atony b. Uterine inversion c. Vaginal hematoma d. Vaginal laceration

ANS: A Uterine atony is significant hypotonia of the uterus and is the leading cause of postpartum hemorrhage. Uterine inversion may lead to hemorrhage; however, it is not the most likely source of this clients bleeding. Further, if the woman were experiencing a uterine inversion, it would be evidenced by the presence of a large, red, rounded mass protruding from the introitus. A vaginal hematoma may be associated with hemorrhage. However, the most likely clinical finding for vaginal hematoma is pain, not the presence of profuse bleeding. A vaginal laceration should be suspected if vaginal bleeding continues in the presence of a firm, contracted uterine fundus.

The reported incidence of ectopic pregnancy has steadily risen over the past 2 decades. Causes include the increase in sexually transmitted infections (STIs) accompanied by tubal infection and damage. The popularity of contraceptive devices such as the IUD has also increased the risk for ectopic pregnancy. The nurse suspects that a client has early signs of ectopic pregnancy. The nurse should be observing the client for which signs or symptoms? (Select all that apply.) a. Pelvic pain b. Abdominal pain c. Unanticipated heavy bleeding d. Vaginal spotting or light bleeding e. Missed period

ANS: A, B, D, E A missed period or spotting can be easily mistaken by the client as an early sign of pregnancy. More subtle signs depend on exactly where the implantation occurs. The nurse must be thorough in her assessment because pain is not a normal symptom of early pregnancy. As the fallopian tube tears open and the embryo is expelled, the client often exhibits severe pain accompanied by intraabdominal hemorrhage, which may progress to hypovolemic shock with minimal or even no external bleeding. In approximately one half of women, shoulder and neck pain results from irritation of the diaphragm from the hemorrhage.

Diabetes refers to a group of metabolic diseases characterized by hyperglycemia resulting from defects in insulin action, insulin secretion, or both. Over time, diabetes causes significant changes in the microvascular and macrovascular circulations. What do these complications include? (Select all that apply.) a. Atherosclerosis b. Retinopathy c. Intrauterine fetal death (IUFD) d. Nephropathy e. Neuropathy f. Autonomic neuropathy

ANS: A, B, D, E These structural changes will most likely affect a variety of systems, including the heart, eyes, kidneys, and nerves. IUFD (stillbirth) remains a major complication of diabetes in pregnancy; however, this is a fetal complication.

One of the most important components of the physical assessment of the pregnant client is the determination of BP. Consistency in measurement techniques must be maintained to ensure that the nuances in the variations of the BP readings are not the result of provider error. Which techniques are important in obtaining accurate BP readings? (Select all that apply.) a. The client should be seated. b. The clients arm should be placed at the level of the heart. c. An electronic BP device should be used. d. The cuff should cover a minimum of 60% of the upper arm. e. The same arm should be used for every reading.

ANS: A, B, E BP readings are easily affected by maternal position. Ideally, the client should be seated. An alternative position is left lateral recumbent with the arm at the level of the heart. The arm should always be held in a horizontal position at approximately the level of the heart. The same arm should be used at every visit. The manual sphygmomanometer is the most accurate device. If manual and electronic devices are used in the care setting, then the nurse must use caution when interpreting the readings. A proper size cuff should cover at least 80% of the upper arm or be approximately 1.5 times the length of the upper arm.

Approximately 10% to 15% of all clinically recognized pregnancies end in miscarriage. What are possible causes of early miscarriage? (Select all that apply.) a. Chromosomal abnormalities b. Infections c. Endocrine imbalance d. Systemic disorders e. Varicella

ANS: A, C, D, E Infections are not a common cause of early miscarriage. At least 50% of pregnancy losses result from chromosomal abnormalities. Endocrine imbalances such as hypothyroidism or diabetes are also possible causes for early pregnancy loss. Other systemic disorders that may contribute to pregnancy loss include lupus and genetic conditions. Although infections are not a common cause of early miscarriage, varicella infection in the first trimester has been associated with pregnancy loss.

The induction of labor is considered an acceptable obstetric procedure if it is in the best interest to deliver the fetus. The charge nurse on the labor and delivery unit is often asked to schedule clients for this procedure and therefore must be cognizant of the specific conditions appropriate for labor induction. What are appropriate indications for induction? (Select all that apply?) a. Rupture of membranes at or near term b. Convenience of the woman or her physician c. Chorioamnionitis (inflammation of the amniotic sac) d. Postterm pregnancy e. Fetal death

ANS: A, C, D, E The conditions listed are all acceptable indications for induction. Other conditions include intrauterine growth restriction (IUGR), maternal-fetal blood incompatibility, hypertension, and placental abruption. Elective inductions for the convenience of the woman or her provider are not recommended; however, they have become commonplace. Factors such as rapid labors and living a long distance from a health care facility may be valid reasons in such a circumstance. Elective delivery should not occur before 39 weeks of completed gestation.

A client who has undergone a D&C for early pregnancy loss is likely to be discharged the same day. The nurse must ensure that her vital signs are stable, that bleeding has been controlled, and that the woman has adequately recovered from the administration of anesthesia. To promote an optimal recovery, what information should discharge teaching include? (Select all that apply.) a. Iron supplementation b. Resumption of intercourse at 6 weeks postprocedure c. Referral to a support group, if necessary d. Expectation of heavy bleeding for at least 2 weeks e. Emphasizing the need for rest

ANS: A, C, E The woman should be advised to consume a diet high in iron and protein. For many women, iron supplementation also is necessary. The nurse should acknowledge that the client has experienced a loss, however early. She can be taught to expect mood swings and possibly depression. Referral to a support group, clergy, or professional counseling may be necessary. Discharge teaching should emphasize the need for rest. Nothing should be placed in the vagina for 2 weeks after the procedure, including tampons and vaginal intercourse. The purpose of this recommendation is to prevent infection. Should infection occur, antibiotics may be prescribed. The client should expect a scant, dark discharge for 1 to 2 weeks. Should heavy, profuse, or bright bleeding occur, she should be instructed to contact her health care provider.

The nurse who elects to work in the specialty of obstetric care must have the ability to distinguish between preterm birth, preterm labor, and low birth weight. Which statement regarding this terminology is correct? a. Terms preterm birth and low birth weight can be used interchangeably. b. Preterm labor is defined as cervical changes and uterine contractions occurring between 20 and 37 weeks of gestation. c. Low birth weight is a newborn who weighs below 3.7 pounds. d. Preterm birth rate in the United States continues to increase.

ANS: B Before 20 weeks of gestation, the fetus is not viable (miscarriage); after 37 weeks, the fetus can be considered term. Although these terms are used interchangeably, they have different meanings: preterm birth describes the length of gestation (before 37 weeks), regardless of the newborns weight; low birth weight describes only the infants weight at the time of birth (2500 g or less), whenever it occurs. Low birth weight is anything below 2500 g or approximately pounds. In 2011, the preterm birth rate in the United States was 11.7 %; it has dropped every year since 2008.

An 18-year-old client who has reached 16 weeks of gestation was recently diagnosed with pregestational diabetes. She attends her centering appointment accompanied by one of her girlfriends. This young woman appears more concerned about how her pregnancy will affect her social life than her recent diagnosis of diabetes. A number of nursing diagnoses are applicable to assist in planning adequate care. What is the most appropriate diagnosis at this time? a. Risk for injury, to the fetus related to birth trauma b. Deficient knowledge, related to diabetic pregnancy management c. Deficient knowledge, related to insulin administration d. Risk for injury, to the mother related to hypoglycemia or hyperglycemia

ANS: B Before a treatment plan is developed or goals for the outcome of care are outlined, this client must come to an understanding of diabetes and the potential effects on her pregnancy. She appears more concerned about changes to her social life than adopting a new self-care regimen. Risk for injury to the fetus related to either placental insufficiency or birth trauma may come later in the pregnancy. At this time, the client is having difficulty acknowledging the adjustments that she needs to make to her lifestyle to care for herself during pregnancy. The client may not yet be on insulin. Insulin requirements increase with gestation. The importance of glycemic control must be part of health teaching for this client. However, she has not yet acknowledged that changes to her lifestyle need to be made and may not participate in the plan of care until understanding takes place.

A perinatal nurse is giving discharge instructions to a woman, status postsuction, and curettage secondary to a hydatidiform mole. The woman asks why she must take oral contraceptives for the next 12 months. What is the bestresponse by the nurse? a. If you get pregnant within 1 year, the chance of a successful pregnancy is very small. Therefore, if you desire a future pregnancy, it would be better for you to use the most reliable method of contraception available. b. The major risk to you after a molar pregnancy is a type of cancer that can be diagnosed only by measuring the same hormone that your body produces during pregnancy. If you were to get pregnant, then it would make the diagnosis of this cancer more difficult. c. If you can avoid a pregnancy for the next year, the chance of developing a second molar pregnancy is rare. Therefore, to improve your chance of a successful pregnancy, not getting pregnant at this time is best. d. Oral contraceptives are the only form of birth control that will prevent a recurrence of a molar pregnancy.

ANS: B Betahuman chorionic gonadotropin (beta-hCG) hormone levels are drawn for 1 year to ensure that the mole is completely gone. The chance of developing choriocarcinoma after the development of a hydatidiform mole is increased. Therefore, the goal is to achieve a zero human chorionic gonadotropin (hCG) level. If the woman were to become pregnant, then it may obscure the presence of the potentially carcinogenic cells. Women should be instructed to use birth control for 1 year after treatment for a hydatidiform mole. The rationale for avoiding pregnancy for 1 year is to ensure that carcinogenic cells are not present. Any contraceptive method except an intrauterine device (IUD) is acceptable.

A client with maternal phenylketonuria (PKU) has come to the obstetrical clinic to begin prenatal care. Why would this preexisting condition result in the need for closer monitoring during pregnancy? a. PKU is a recognized cause of preterm labor. b. The fetus may develop neurologic problems. c. A pregnant woman is more likely to die without strict dietary control. d. Women with PKU are usually mentally handicapped and should not reproduce.

ANS: B Children born to women with untreated PKU are more likely to be born with mental retardation, microcephaly, congenital heart disease, and low birth weight. Maternal PKU has no effect on labor. Women without dietary control of PKU are more likely to miscarry or bear a child with congenital anomalies. Screening for undiagnosed maternal PKU at the first prenatal visit may be warranted, especially in individuals with a family history of the disorder, with low intelligence of an uncertain cause, or who have given birth to microcephalic infants.

Which laboratory marker is indicative of DIC? a. Bleeding time of 10 minutes b. Presence of fibrin split products c. Thrombocytopenia d. Hypofibrinogenemia

ANS: B Degradation of fibrin leads to the accumulation of multiple fibrin clots throughout the bodys vasculature. Bleeding time in DIC is normal. Low platelets may occur but are not indicative of DIC because they may be the result from other coagulopathies. Hypofibrinogenemia occurs with DIC.

Which maternal condition always necessitates delivery by cesarean birth? a. Marginal placenta previa b. Complete placenta previa c. Ectopic pregnancy d. Eclampsia

ANS: B In complete placenta previa, the placenta completely covers the cervical os. A cesarean birth is the acceptable method of delivery. The risk of fetal death occurring is due to preterm birth. If the previa is marginal (i.e., 2 cm or greater away from the cervical os), then labor can be attempted. A cesarean birth is not indicated for an ectopic pregnancy. Labor can be safely induced if the eclampsia is under control.

During an inpatient psychiatric hospitalization, what is the most important nursing intervention? a. Contacting the clients significant other b. Supervising and guiding visits with her infant c. Allowing no contact with anyone who annoys her d. Having the infant with the mother at all times

ANS: B In the hospital setting, the reintroduction of the infant to the mother can and should occur at the mothers own pace. A schedule is set that increases the number of hours the mother cares for her infant over several days, culminating in the infant staying overnight in the mothers room. These supervised and guided visits allow the mother to experience meeting the infants needs and giving up sleep for the infant. Reintroducing the mother to her infant while in a supervised setting is essential. Another important task for a mother under psychiatric care is to reestablish positive interactions with others.

The nurse who is caring for a woman hospitalized for hyperemesis gravidarum would expect the initial treatment to involve what? a. Corticosteroids to reduce inflammation b. Intravenous (IV) therapy to correct fluid and electrolyte imbalances c. Antiemetic medication, such as pyridoxine, to control nausea and vomiting d. Enteral nutrition to correct nutritional deficits

ANS: B Initially, the woman who is unable to down clear liquids by mouth requires IV therapy to correct fluid and electrolyte imbalances. Corticosteroids have been successfully used to treat refractory hyperemesis gravidarum, but they are not the expected initial treatment for this disorder. Pyridoxine is vitamin B6, not an antiemetic medication. Promethazine, a common antiemetic, may be prescribed. In severe cases of hyperemesis gravidarum, enteral nutrition via a feeding tube may be necessary to correct maternal nutritional deprivation but is not the initial treatment for this client.

Which client is at greatest risk for early PPH? a. Primiparous woman (G 2, P 1-0-0-1) being prepared for an emergency cesarean birth for fetal distress b. Woman with severe preeclampsia on magnesium sulfate whose labor is being induced c. Multiparous woman (G 3, P 2-0-0-2) with an 8-hour labor d. Primigravida in spontaneous labor with preterm twins

ANS: B Magnesium sulfate administration during labor poses a risk for PPH. Magnesium acts as a smooth muscle relaxant, thereby contributing to uterine relaxation and atony. A primiparous woman being prepared for an emergency cesarean birth for fetal distress, a multiparous woman with an 8-hour labor, and a primigravida in spontaneous labor with preterm twins do not indicate risk factors or causes of early PPH.

The use of methamphetamine (meth) has been described as a significant drug problem in the United States. The nurse who provides care to this client population should be cognizant of what regarding methamphetamine use? a. Methamphetamines are similar to opiates. b. Methamphetamines are stimulants with vasoconstrictive characteristics. c. Methamphetamines should not be discontinued during pregnancy. d. Methamphetamines are associated with a low rate of relapse.

ANS: B Methamphetamines are stimulants with vasoconstrictive characteristics similar to cocaine and are similarly used. As is the case with cocaine users, methamphetamine users are urged to immediately stop all use during pregnancy. Unfortunately, because methamphetamine users are extremely psychologically addicted, the rate of relapse is extremely high.

What is the correct terminology for an abortion in which the fetus dies but is retained within the uterus? a. Inevitable abortion b. Missed abortion c. Incomplete abortion d. Threatened abortion

ANS: B Missed abortion refers to the retention of a dead fetus in the uterus. An inevitable abortion means that the cervix is dilating with the contractions. An incomplete abortion means that not all of the products of conception were expelled. With a threatened abortion, the woman has cramping and bleeding but no cervical dilation.

A pregnant woman is being discharged from the hospital after the placement of a cervical cerclage because of a history of recurrent pregnancy loss, secondary to an incompetent cervix. Which information regarding postprocedural care should the nurse emphasize in the discharge teaching? a. Any vaginal discharge should be immediately reported to her health care provider. b. The presence of any contractions, rupture of membranes (ROM), or severe perineal pressure should be reported. c. The client will need to make arrangements for care at home, because her activity level will be restricted. d. The client will be scheduled for a cesarean birth.

ANS: B Nursing care should stress the importance of monitoring for the signs and symptoms of preterm labor. Vaginal bleeding needs to be reported to her primary health care provider. Bed rest is an element of care. However, the woman may stand for periods of up to 90 minutes, which allows her the freedom to see her physician. Home uterine activity monitoring may be used to limit the womans need for visits and to monitor her status safely at home. The cerclage can be removed at 37 weeks of gestation (to prepare for a vaginal birth), or a cesarean birth can be planned.

A new mother with a thyroid disorder has come for a lactation follow-up appointment. Which thyroid disorder is a contraindication for breastfeeding? a. Hyperthyroidism b. PKU c. Hypothyroidism d. Thyroid storm

ANS: B PKU is a cause of mental retardation in infants; mothers with PKU pass on phenylalanine and therefore should elect not to breastfeed. A woman with either hyperthyroidism or hypothyroidism would have no particular reason not to breastfeed. A thyroid storm is a complication of hyperthyroidism and is not a contraindication to breastfeeding.

Preconception counseling is critical in the safe management of diabetic pregnancies. Which complication is commonly associated with poor glycemic control before and during early pregnancy? a. Frequent episodes of maternal hypoglycemia b. Congenital anomalies in the fetus c. Hydramnios d. Hyperemesis gravidarum

ANS: B Preconception counseling is particularly important since strict metabolic control before conception and in the early weeks of gestation is instrumental in decreasing the risk of congenital anomalies. Frequent episodes of maternal hypoglycemia may occur during the first trimester (not before conception) as a result of hormonal changes and the effects on insulin production and use. Hydramnios occurs approximately 10 times more often in diabetic pregnancies than in nondiabetic pregnancies. Typically, it is observed in the third trimester of pregnancy. Hyperemesis gravidarum may exacerbate hypoglycemic events because the decreased food intake by the mother and glucose transfer to the fetus contribute to hypoglycemia.

Which nursing intervention is paramount when providing care to a client with preterm labor who has received terbutaline? a. Assess deep tendon reflexes (DTRs). b. Assess for dyspnea and crackles. c. Assess for bradycardia. d. Assess for hypoglycemia.

ANS: B Terbutaline is a beta2-adrenergic agonist that affects the mothers cardiopulmonary and metabolic systems. Signs of cardiopulmonary decompensation include adventitious breath sounds and dyspnea. An assessment for dyspnea and crackles is important for the nurse to perform if the woman is taking magnesium sulfate. Assessing DTRs does not address the possible respiratory side effects of using terbutaline. Since terbutaline is a beta2-adrenergic agonist, it can lead to hyperglycemia, not hypoglycemia. Beta2-adrenergic agonist drugs cause tachycardia, not bradycardia.

A 26-year-old pregnant woman, gravida 2, para 1-0-0-1, is 28 weeks pregnant when she experiences bright red, painless vaginal bleeding. On her arrival at the hospital, which diagnostic procedure will the client most likely have performed? a. Amniocentesis for fetal lung maturity b. Transvaginal ultrasound for placental location c. Contraction stress test (CST) d. Internal fetal monitoring

ANS: B The presence of painless bleeding should always alert the health care team to the possibility of placenta previa, which can be confirmed through ultrasonography. Amniocentesis is not performed on a woman who is experiencing bleeding. In the event of an imminent delivery, the fetus is presumed to have immature lungs at this gestational age, and the mother is given corticosteroids to aid in fetal lung maturity. A CST is not performed at a preterm gestational age. Furthermore, bleeding is a contraindication to a CST. Internal fetal monitoring is also contraindicated in the presence of bleeding.

A 26-year-old primigravida has come to the clinic for her regular prenatal visit at 12 weeks. She appears thin and somewhat nervous. She reports that she eats a well-balanced diet, although her weight is 5 pounds less than it was at her last visit. The results of laboratory studies confirm that she has a hyperthyroid condition. Based on the available data, the nurse formulates a plan of care. Which nursing diagnosis is most appropriate for the client at this time? a. Deficient fluid volume b. Imbalanced nutrition: less than body requirements c. Imbalanced nutrition: more than body requirements d. Disturbed sleep pattern

ANS: B This clients clinical cues include weight loss, which supports a nursing diagnosis of Imbalanced nutrition: less than body requirements. No clinical signs or symptoms support a nursing diagnosis of deficient fluid volume. This client reports weight loss, not weight gain. Although the client reports nervousness, the most appropriate nursing diagnosis, based on the clients other clinical symptoms, is Imbalanced nutrition: less than body requirements.

A pregnant woman at 29 weeks of gestation has been diagnosed with preterm labor. Her labor is being controlled with tocolytic medications. She asks when she might be able to go home. Which response by the nurse is mostaccurate? a. After the baby is born. b. When we can stabilize your preterm labor and arrange home health visits. c. Whenever your physician says that it is okay. d. It depends on what kind of insurance coverage you have.

ANS: B This clients preterm labor is being controlled with tocolytics. Once she is stable, home care may be a viable option for this type of client. Care of a client with preterm labor is multidisciplinary and multifactorial; the goal is to prevent delivery. In many cases, this goal may be achieved at home. Managed care may dictate an earlier hospital discharge or a shift from hospital to home care. Insurance coverage may be one factor in client care, but ultimately, client safety remains the most important factor.

What would a steady trickle of bright red blood from the vagina in the presence of a firm fundus suggest to the nurse? a. Uterine atony b. Lacerations of the genital tract c. Perineal hematoma d. Infection of the uterus

ANS: B Undetected lacerations will bleed slowly and continuously. Bleeding from lacerations is uncontrolled by uterine contraction. The fundus is not firm in the presence of uterine atony. A hematoma would develop internally. Swelling and discoloration would be noticeable; however, bright bleeding would not be. With an infection of the uterus, an odor to the lochia and systemic symptoms such as fever and malaise would be present.

The nurse recognizes that uterine hyperstimulation with oxytocin requires emergency interventions. What clinical cues alert the nurse that the woman is experiencing uterine hyperstimulation? (Select all that apply.) a. Uterine contractions lasting <90 seconds and occurring >2 minutes in frequency b. Uterine contractions lasting >90 seconds and occurring <2 minutes in frequency c. Uterine tone <20 mm Hg d. Uterine tone >20 mm Hg e. Increased uterine activity accompanied by a nonreassuring FHR and pattern

ANS: B, D, E Uterine contractions that occur less frequently than 2 minutes apart and last longer than 90 seconds, a uterine tone over 20 mm Hg, and a nonreassuring FHR and pattern are indications of uterine hyperstimulation with oxytocin administration. Uterine contractions that occur more frequently than 2 minutes apart and last less than 90 seconds are the expected goal of oxytocin induction. A uterine tone less than 20 mm Hg is normal.

Which preexisting factor is known to increase the risk of GDM? a. Underweight before pregnancy b. Maternal age younger than 25 years c. Previous birth of large infant d. Previous diagnosis of type 2 diabetes mellitus

ANS: C A previous birth of a large infant suggests GDM. Obesity (body mass index [BMI] of 30 or greater) creates a higher risk for gestational diabetes. A woman younger than 25 years is not generally at risk for GDM. The person with type 2 diabetes mellitus already has diabetes and thus will continue to have it after pregnancy. Insulin may be required during pregnancy because oral hypoglycemia drugs are contraindicated during pregnancy.

A woman arrives at the emergency department with complaints of bleeding and cramping. The initial nursing history is significant for a last menstrual period 6 weeks ago. On sterile speculum examination, the primary care provider finds that the cervix is closed. The anticipated plan of care for this woman would be based on a probable diagnosis of which type of spontaneous abortion? a. Incomplete b. Inevitable c. Threatened d. Septic

ANS: C A woman with a threatened abortion has spotting, mild cramps, and no cervical dilation. A woman with an incomplete abortion would have heavy bleeding, mild-to-severe cramping, and cervical dilation. An inevitable abortion demonstrates the same symptoms as an incomplete abortion: heavy bleeding, mild-to-severe cramping, and cervical dilation. A woman with a septic abortion has malodorous bleeding and typically a dilated cervix.

A woman with worsening preeclampsia is admitted to the hospitals labor and birth unit. The physician explains the plan of care for severe preeclampsia, including the induction of labor, to the woman and her husband. Which statement by the husband leads the nurse to believe that the couple needs further information? a. I will help my wife use the breathing techniques that we learned in our childbirth classes. b. I will give my wife ice chips to eat during labor. c. Since we will be here for a while, I will call my mother so she can bring the two boys2 years and 4 years of ageto visit their mother. d. I will stay with my wife during her labor, just as we planned.

ANS: C Arranging a visit with their two children indicates that the husband does not understand the importance of the quiet, subdued environment that is needed to prevent his wifes condition from worsening. Implementing breathing techniques is indicative of adequate knowledge related to pain management during labor. Administering ice chips indicates an understanding of nutritional needs during labor. Staying with his wife during labor demonstrates the husbands support for his wife and is appropriate.

What is the primary purpose for the use of tocolytic therapy to suppress uterine activity? a. Drugs can be efficaciously administered up to the designated beginning of term at 37 weeks gestation. b. Tocolytic therapy has no important maternal (as opposed to fetal) contraindications. c. The most important function of tocolytic therapy is to provide the opportunity to administer antenatal glucocorticoids. d. If the client develops pulmonary edema while receiving tocolytic therapy, then intravenous (IV) fluids should be given.

ANS: C Buying time for antenatal glucocorticoids to accelerate fetal lung development may be the best reason to use tocolytic therapy. Once the pregnancy has reached 34 weeks, however, the risks of tocolytic therapy outweigh the benefits. Important maternal contraindications to tocolytic therapy exist. Tocolytic-induced edema can be caused by IV fluids.

What condition indicates concealed hemorrhage when the client experiences abruptio placentae? a. Decrease in abdominal pain b. Bradycardia c. Hard, boardlike abdomen d. Decrease in fundal height

ANS: C Concealed hemorrhage occurs when the edges of the placenta do not separate. The formation of a hematoma behind the placenta and subsequent infiltration of the blood into the uterine muscle results in a very firm, boardlike abdomen. Abdominal pain may increase. The client will have shock symptoms that include tachycardia. As bleeding occurs, the fundal height increases.

A woman arrives for evaluation of signs and symptoms that include a missed period, adnexal fullness, tenderness, and dark red vaginal bleeding. On examination, the nurse notices an ecchymotic blueness around the womans umbilicus. What does this finding indicate? a. Normal integumentary changes associated with pregnancy b. Turner sign associated with appendicitis c. Cullen sign associated with a ruptured ectopic pregnancy d. Chadwick sign associated with early pregnancy

ANS: C Cullen sign, the blue ecchymosis observed in the umbilical area, indicates hematoperitoneum associated with an undiagnosed ruptured intraabdominal ectopic pregnancy. Linea nigra on the abdomen is the normal integumentary change associated with pregnancy and exhibits a brown pigmented, vertical line on the lower abdomen. Turner sign is ecchymosis in the flank area, often associated with pancreatitis. A Chadwick sign is a blue-purple cervix that may be seen during or around the eighth week of pregnancy.

During a prenatal visit, the nurse is explaining dietary management to a woman with pregestational diabetes. Which statement by the client reassures the nurse that teaching has been effective? a. I will need to eat 600 more calories per day because I am pregnant. b. I can continue with the same diet as before pregnancy as long as it is well balanced. c. Diet and insulin needs change during pregnancy. d. I will plan my diet based on the results of urine glucose testing.

ANS: C Diet and insulin needs change during the pregnancy in direct correlation to hormonal changes and energy needs. In the third trimester, insulin needs may double or even quadruple. The diet is individualized to allow for increased fetal and metabolic requirements, with consideration of such factors as prepregnancy weight and dietary habits, overall health, ethnic background, lifestyle, stage of pregnancy, knowledge of nutrition, and insulin therapy. Energy needs are usually calculated on the basis of 30 to 35 calories per kilogram of ideal body weight. Dietary management during a diabetic pregnancy must be based on blood, not urine, glucose changes.

The labor of a pregnant woman with preeclampsia is going to be induced. Before initiating the oxytocin (Pitocin) infusion, the nurse reviews the womans latest laboratory test findings, which reveal a platelet count of 90,000 mm3, an elevated aspartate aminotransaminase (AST) level, and a falling hematocrit. The laboratory results are indicative of which condition? a. Eclampsia b. Disseminated intravascular coagulation (DIC) syndrome c. Hemolysis, elevated liver enzyme levels, and low platelet levels (HELLP) syndrome d. Idiopathic thrombocytopenia

ANS: C HELLP syndrome is a laboratory diagnosis for a variant of severe preeclampsia that involves hepatic dysfunction characterized by hemolysis (H), elevated liver (EL) enzymes, and low platelets (LP). Eclampsia is determined by the presence of seizures. DIC is a potential complication associated with HELLP syndrome. Idiopathic thrombocytopenia is the presence of low platelets of unknown cause and is not associated with preeclampsia.

To manage her diabetes appropriately and to ensure a good fetal outcome, how would the pregnant woman with diabetes alter her diet? a. Eat six small equal meals per day. b. Reduce the carbohydrates in her diet. c. Eat her meals and snacks on a fixed schedule. d. Increase her consumption of protein.

ANS: C Having a fixed meal schedule will provide the woman and the fetus with a steady blood sugar level, provide a good balance with insulin administration, and help prevent complications. Having a fixed meal schedule is more important than the equal division of food intake. Approximately 45% of the food eaten should be in the form of carbohydrates.

Which statement regarding the laboratory test for glycosylated hemoglobin Alc is correct? a. The laboratory test for glycosylated hemoglobin Alc is performed for all pregnant women, not only those with or likely to have diabetes. b. This laboratory test is a snapshot of glucose control at the moment. c. This laboratory test measures the levels of hemoglobin Alc, which should remain at less than 7%. d. This laboratory test is performed on the womans urine, not her blood.

ANS: C Hemoglobin Alc levels greater than 7% indicate an elevated glucose level during the previous 4 to 6 weeks. This extra laboratory test is for diabetic women and defines glycemic control over the previous 4 to 6 weeks. Glycosylated hemoglobin level tests are performed on the blood.

What is the primary purpose for magnesium sulfate administration for clients with preeclampsia and eclampsia? a. To improve patellar reflexes and increase respiratory efficiency b. To shorten the duration of labor c. To prevent convulsions d. To prevent a boggy uterus and lessen lochial flow

ANS: C Magnesium sulfate is the drug of choice used to prevent convulsions, although it can generate other problems. Loss of patellar reflexes and respiratory depression are signs of magnesium toxicity. Magnesium sulfate can also increase the duration of labor. Women are at risk for a boggy uterus and heavy lochial flow as a result of magnesium sulfate therapy.

The nurse is preparing to administer methotrexate to the client. This hazardous drug is most often used for which obstetric complication? a. Complete hydatidiform mole b. Missed abortion c. Unruptured ectopic pregnancy d. Abruptio placentae

ANS: C Methotrexate is an effective nonsurgical treatment option for a hemodynamically stable woman whose ectopic pregnancy is unruptured and measures less than 4 cm in diameter. Methotrexate is not indicated or recommended as a treatment option for a complete hydatidiform mole, for a missed abortion, or for abruptio placentae.

Women with mild gestational hypertension and mild preeclampsia can be safely managed at home with frequent maternal and fetal evaluation. Complete or partial bed rest is still frequently ordered by some providers. Which complication is rarely the result of prolonged bed rest? a. Thrombophlebitis b. Psychologic stress c. Fluid retention d. Cardiovascular deconditioning

ANS: C No evidence has been found that supports the practice of bed rest to improve pregnancy outcome. Fluid retention is not an adverse outcome of prolonged bed rest. The woman is more likely to experience diuresis with accompanying fluid and electrolyte imbalance and weight loss. Prolonged bed rest is known to increase the risk for thrombophlebitis. Psychologic stress is known to begin on the first day of bed rest and continue for the duration of the therapy. Therefore, restricted activity, rather than complete bed rest, is recommended. Cardiovascular deconditioning is a known complication of bed rest.

A woman at 24 weeks of gestation states that she has a glass of wine with dinner every evening. Why would the nurse counsel the client to eliminate all alcohol? a. Daily consumption of alcohol indicates a risk for alcoholism. b. She will be at risk for abusing other substances as well. c. The fetus is placed at risk for altered brain growth. d. The fetus is at risk for multiple organ anomalies.

ANS: C No period exists when consuming alcohol during pregnancy is safe. The documented effects of alcohol consumption during pregnancy include mental retardation, learning disabilities, high activity level, and short attention span. The brain grows most rapidly in the third trimester and is vulnerable to alcohol exposure during this time. Abuse of other substances has not been linked to alcohol use.

The management of the pregnant client who has experienced a pregnancy loss depends on the type of miscarriage and the signs and symptoms. While planning care for a client who desires outpatient management after a first-trimester loss, what would the nurse expect the plan to include? a. Dilation and curettage (D&C) b. Dilation and evacuation (D&E) c. Misoprostol d. Ergot products

ANS: C Outpatient management of a first-trimester loss is safely accomplished by the intravaginal use of misoprostol for up to 2 days. If the bleeding is uncontrollable, vital signs are unstable, or signs of infection are present, then a surgical evacuation should be performed. D&C is a surgical procedure that requires dilation of the cervix and scraping of the uterine walls to remove the contents of pregnancy. This procedure is commonly performed to treat inevitable or incomplete abortion and should be performed in a hospital. D&E is usually performed after 16 weeks of pregnancy. The cervix is widely dilated, followed by removal of the contents of the uterus. Ergot products such as Methergine or Hemabate may be administered for excessive bleeding after miscarriage.

While providing care to the maternity client, the nurse should be aware that one of these anxiety disorders is likely to be triggered by the process of labor and birth. Which disorder fits this criterion? a. Phobias b. Panic disorder c. Posttraumatic stress disorder (PTSD) d. Obsessive-compulsive disorder (OCD)

ANS: C PTSD can occur as the result of a past trauma such as rape. Symptoms of PTSD include re-experiencing the event, numbing, irritability, angry outbursts, and exaggerated startle reflex. With the increased bodily touch and vaginal examinations that occur during labor, the client may have memories of the original trauma. The process of giving birth may result in her feeling out of control. The nurse should verbalize an understanding and reassure the client as necessary. Phobias are irrational fears that may lead a person to avoid certain events or situations. Panic disorders may occur in as many as 3% to 5% of women in the postpartum period and are described as episodes of intense apprehension, fear, and terror. Symptoms of a panic disorder may include palpitations, chest pain, choking, or smothering. OCD symptoms include recurrent, persistent, and intrusive thoughts. The mother may repeatedly check and recheck her infant once he or she is born, although she realizes that this behavior is irrational. OCD is optimally treated with medications.

A woman who has recently given birth complains of pain and tenderness in her leg. On physical examination, the nurse notices warmth and redness over an enlarged, hardened area. Which condition should the nurse suspect, and how will it be confirmed? a. Disseminated intravascular coagulation (DIC); asking for laboratory tests b. von Willebrand disease (vWD); noting whether bleeding times have been extended c. Thrombophlebitis; using real-time and color Doppler ultrasound d. Idiopathic or immune thrombocytopenic purpura (ITP); drawing blood for laboratory analysis

ANS: C Pain and tenderness in the extremities, which show warmth, redness, and hardness, is likely thrombophlebitis. A Doppler ultrasound examination is a common noninvasive way to confirm the diagnosis. A diagnosis of DIC is made according to clinical findings and laboratory markers. With DIC, a physical examination will reveal symptoms that may include unusual bleeding, petechiae around a blood pressure cuff on the womans arm, and/or excessive bleeding from the site of a slight trauma such as a venipuncture site. Symptoms of vWD, a type of hemophilia, include recurrent bleeding episodes, prolonged bleeding time, and factor VIII deficiency. A risk for PPH exists with vWD but does not exhibit a warm or reddened area in an extremity. ITP is an autoimmune disorder in which the life span of antiplatelet antibodies is decreased. Increased bleeding time is a diagnostic finding, and the risk of postpartum uterine bleeding is increased.

A number of metabolic changes occur throughout pregnancy. Which physiologic adaptation of pregnancy will influence the nurses plan of care? a. Insulin crosses the placenta to the fetus only in the first trimester, after which the fetus secretes its own. b. Women with insulin-dependent diabetes are prone to hyperglycemia during the first trimester because they are consuming more sugar. c. During the second and third trimesters, pregnancy exerts a diabetogenic effect that ensures an abundant supply of glucose for the fetus. d. Maternal insulin requirements steadily decline during pregnancy.

ANS: C Pregnant women develop increased insulin resistance during the second and third trimesters. Insulin never crosses the placenta; the fetus starts making its own around the 10th week. As a result of normal metabolic changes during pregnancy, insulin-dependent women are prone to hypoglycemia (low levels). Maternal insulin requirements may double or quadruple by the end of pregnancy.

Prostaglandin gel has been ordered for a pregnant woman at 43 weeks of gestation. What is the primary purpose of prostaglandin administration? a. To enhance uteroplacental perfusion in an aging placenta b. To increase amniotic fluid volume c. To ripen the cervix in preparation for labor induction d. To stimulate the amniotic membranes to rupture

ANS: C Preparations of prostaglandin E1 and E2 are effective when used before labor induction to ripen (i.e., soften and thin) the cervix. Uteroplacental perfusion is not altered by the use of prostaglandins. The insertion of prostaglandin gel has no effect on the level of amniotic fluid. In some cases, women will spontaneously begin laboring after the administration of prostaglandins, thereby eliminating the need for oxytocin. It is not common for a womans membranes to rupture as a result of prostaglandin use.

In planning for home care of a woman with preterm labor, which concern should the nurse need to address? a. Nursing assessments are different from those performed in the hospital setting. b. Restricted activity and medications are necessary to prevent a recurrence of preterm labor. c. Prolonged bed rest may cause negative physiologic effects. d. Home health care providers are necessary.

ANS: C Prolonged bed rest may cause adverse effects such as weight loss, loss of appetite, muscle wasting, weakness, bone demineralization, decreased cardiac output, risk for thrombophlebitis, alteration in bowel functions, sleep disturbance, and prolonged postpartum recovery. Nursing assessments differ somewhat from those performed in the acute care setting, but this concern does not need to be addressed. Restricted activity and medications may prevent preterm labor but not in all women. In addition, the plan of care is individualized to meet the needs of each client. Many women receive home health nurse visits, but care is individualized for each woman.

A primigravida is being monitored at the prenatal clinic for preeclampsia. Which finding is of greatest concern to the nurse? a. Blood pressure (BP) increase to 138/86 mm Hg b. Weight gain of 0.5 kg during the past 2 weeks c. Dipstick value of 3+ for protein in her urine d. Pitting pedal edema at the end of the day

ANS: C Proteinuria is defined as a concentration of 1+ or greater via dipstick measurement. A dipstick value of 3+ alerts the nurse that additional testing or assessment should be performed. A 24-hour urine collection is preferred over dipstick testing attributable to accuracy. Generally, hypertension is defined as a BP of 140/90 mm Hg or an increase in systolic pressure of 30 mm Hg or diastolic pressure of 15 mm Hg. Preeclampsia may be demonstrated as a rapid weight gain of more than 2 kg in 1 week. Edema occurs in many normal pregnancies, as well as in women with preeclampsia. Therefore, the presence of edema is no longer considered diagnostic of preeclampsia.

A pregnant woman has been receiving a magnesium sulfate infusion for treatment of severe preeclampsia for 24 hours. On assessment, the nurse finds the following vital signs: temperature 37.3 C, pulse rate 88 beats per minute, respiratory rate 10 breaths per minute, BP 148/90 mm Hg, absent deep tendon reflexes (DTRs), and no ankle clonus. The client complains, Im so thirsty and warm. What is the nurses immediate action? a. To call for an immediate magnesium sulfate level b. To administer oxygen c. To discontinue the magnesium sulfate infusion d. To prepare to administer hydralazine

ANS: C Regardless of the magnesium level, the client is displaying the clinical signs and symptoms of magnesium toxicity. The first action by the nurse should be to discontinue the infusion of magnesium sulfate. In addition, calcium gluconate, the antidote for magnesium, may be administered. Hydralazine is an antihypertensive drug commonly used to treat hypertension in severe preeclampsia. Typically, hydralazine is administered for a systolic BP higher than 160 mm Hg or a diastolic BP higher than 110 mm Hg.

The nurse is teaching a client with preterm premature rupture of membranes (PPROM) regarding self-care activities. Which activities should the nurse include in her teaching? a. Report a temperature higher than 40 C. b. Tampons are safe to use to absorb the leaking amniotic fluid. c. Do not engage in sexual activity. d. Taking frequent tub baths is safe.

ANS: C Sexual activity should be avoided because it may induce preterm labor. A temperature higher than 38 C should be reported. To prevent the risk of infection, tub baths should be avoided and nothing should be inserted into the vagina. Further, foul-smelling vaginal fluid, which may be a sign of infection, should be reported.

Which substance used during pregnancy causes vasoconstriction and decreased placental perfusion, resulting in maternal and neonatal complications? a. Alcohol b. Caffeine c. Tobacco d. Chocolate

ANS: C Smoking in pregnancy is known to cause a decrease in placental perfusion and is the cause of low-birth-weight infants. Prenatal alcohol exposure is the single greatest preventable cause of mental retardation. Alcohol use during pregnancy can cause high blood pressure, miscarriage, premature birth, stillbirth, and anemia. Caffeine may interfere with certain medications and worsen arrhythmias. Chocolate, particularly dark chocolate, contains caffeine that may interfere with certain medications.

A woman who is 30 weeks of gestation arrives at the hospital with bleeding. Which differential diagnosis would not be applicable for this client? a. Placenta previa b. Abruptio placentae c. Spontaneous abortion d. Cord insertion

ANS: C Spontaneous abortion is another name for miscarriage; it occurs, by definition, early in pregnancy. Placenta previa is a well-known reason for bleeding late in pregnancy. The premature separation of the placenta (abruptio placentae) is a bleeding disorder that can occur late in pregnancy. Cord insertion may cause a bleeding disorder that can also occur late in pregnancy.

The most effective and least expensive treatment of puerperal infection is prevention. What is the most important strategy for the nurse to adopt? a. Large doses of vitamin C during pregnancy b. Prophylactic antibiotics c. Strict aseptic technique, including hand washing, by all health care personnel d. Limited protein and fat intake

ANS: C Strict adherence by all health care personnel to aseptic techniques during childbirth and the postpartum period is extremely important and the least expensive measure to prevent infection. Good nutrition to control anemia is a preventive measure. Increased iron intake assists in preventing anemia. Antibiotics may be administered to manage infections; they are not a cost-effective measure to prevent postpartum infection. Limiting protein and fat intake does not help prevent anemia or prevent infection.

Which statement most accurately describes the HELLP syndrome? a. Mild form of preeclampsia b. Diagnosed by a nurse alert to its symptoms c. Characterized by hemolysis, elevated liver enzymes, and low platelets d. Associated with preterm labor but not perinatal mortality

ANS: C The acronym HELLP stands for hemolysis (H), elevated liver (EL) enzymes, and low platelets (LP). The HELLP syndrome is a variant of severe preeclampsia and is difficult to identify because the symptoms are not often obvious. The HELLP syndrome must be diagnosed in the laboratory. Preterm labor is greatly increased; therefore, so is perinatal mortality.

A primigravida at 40 weeks of gestation is having uterine contractions every to 2 minutes and states that they are very painful. Her cervix is dilated 2 cm and has not changed in 3 hours. The woman is crying and wants an epidural. What is the likely status of this womans labor? a. She is exhibiting hypotonic uterine dysfunction. b. She is experiencing a normal latent stage. c. She is exhibiting hypertonic uterine dysfunction. d. She is experiencing precipitous labor.

ANS: C The contraction pattern observed in this woman signifies hypertonic uterine activity. Typically, uterine activity in this phase occurs at 4- to 5-minute intervals lasting 30 to 45 seconds. Women who experience hypertonic uterine dysfunction, or primary dysfunctional labor, are often anxious first-time mothers who are having painful and frequent contractions that are ineffective at causing cervical dilation or effacement to progress. With hypotonic uterine dysfunction, the woman initially makes normal progress into the active stage of labor; then the contractions become weak and inefficient or stop altogether. Precipitous labor is one that lasts less than 3 hours from the onset of contractions until time of birth.

When a woman is diagnosed with postpartum depression (PPD) with psychotic features, what is the nurses primary concern in planning the clients care? a. Displaying outbursts of anger b. Neglecting her hygiene c. Harming her infant d. Losing interest in her husband

ANS: C Thoughts of harm to herself or to the infant are among the most serious symptoms of PPD and require immediate assessment and intervention. Although outbursts of anger and neglecting personal hygiene are symptoms attributable to PPD, the major concern remains the potential of harm to herself or her infant. Although this client is likely to lose interest in her spouse, it is not the nurses primary concern.

The American College of Obstetricians and Gynecologists (ACOG) has developed a comprehensive list of risk factors associated with the development of preeclampsia. Which client exhibits the greatest number of these risk factors? a. 30-year-old obese Caucasian with her third pregnancy b. 41-year-old Caucasian primigravida c. 19-year-old African American who is pregnant with twins d. 25-year-old Asian American whose pregnancy is the result of donor insemination

ANS: C Three risk factors are present in the 19-year-old African-American client. She has African-American ethnicity, is at the young end of the age distribution, and has a multiple pregnancy. In planning care for this client, the nurse must frequently monitor her BP and teach her to recognize the early warning signs of preeclampsia. The 30-year-old obese Caucasian client has only has one known risk factor: obesity. Age distribution appears to be U-shaped, with women younger than 20 years of age and women older than 40 years of age being at greatest risk. Preeclampsia continues to be more frequently observed in primigravidas; this client is a multigravida woman. Two risk factors are present for the 41-year-old Caucasian primigravida client. Her age and status as a primigravida place her at increased risk for preeclampsia. Caucasian women are at a lower risk than are African-American women. The 25-year-old Asian-American client exhibits only one risk factor. Pregnancies that result from donor insemination, oocyte donation, and embryo donation are at an increased risk of developing preeclampsia.

Which statement related to cephalopelvic disproportion (CPD) is the least accurate? a. CPD can be related to either fetal size or fetal position. b. The fetus cannot be born vaginally. c. CPD can be accurately predicted. d. Causes of CPD may have maternal or fetal origins.

ANS: C Unfortunately, accurately predicting CPD is not possible. Although CPD is often related to excessive fetal size (macrosomia), malposition of the fetal presenting part is the problem in many cases, not true CPD. When CPD is present, the fetus cannot fit through the maternal pelvis to be born vaginally. CPD may be related to either fetal origins such as macrosomia or malposition or maternal origins such as a too small or malformed pelvis.

A woman is having her first child. She has been in labor for 15 hours. A vaginal examination performed 2 hours earlier revealed the cervix to be dilated to 5 cm and 100% effaced, and the presenting part of the fetus was at station 0; however, another vaginal examination performed 5 minutes ago indicated no changes. What abnormal labor pattern is associated with this description? a. Prolonged latent phase b. Protracted active phase c. Secondary arrest d. Protracted descent

ANS: C With a secondary arrest of the active phase, the progress of labor has stopped. This client has not had any anticipated cervical change, indicating an arrest of labor. In the nulliparous woman, a prolonged latent phase typically lasts longer than 20 hours. A protracted active phase, the first or second stage of labor, is prolonged (slow dilation). With a protracted descent, the fetus fails to descend at an anticipated rate during the deceleration phase and second stage of labor.

A woman at 26 weeks of gestation is being assessed to determine whether she is experiencing preterm labor. Which finding indicates that preterm labor is occurring? a. Estriol is not found in maternal saliva. b. Irregular, mild uterine contractions are occurring every 12 to 15 minutes. c. Fetal fibronectin is present in vaginal secretions. d. The cervix is effacing and dilated to 2 cm.

ANS: D Cervical changes such as shortened endocervical length, effacement, and dilation are predictors of imminent preterm labor. Changes in the cervix accompanied by regular contractions indicate labor at any gestation. Estriol is a form of estrogen produced by the fetus that is present in plasma at 9 weeks of gestation. Levels of salivary estriol have been shown to increase before preterm birth. Irregular, mild contractions that do not cause cervical change are not considered a threat. The presence of fetal fibronectin in vaginal secretions between 24 and 36 weeks of gestation could predict preterm labor, but it has only a 20% to 40% positive predictive value. Of more importance are other physiologic clues of preterm labor such as cervical changes.

As a powerful central nervous system (CNS) stimulant, which of these substances can lead to miscarriage, preterm labor, placental separation (abruption), and stillbirth? a. Heroin b. Alcohol c. Phencyclidine (1-phenylcyclohexylpiperidine; PCP) d. Cocaine

ANS: D Cocaine is a powerful CNS stimulant. Effects on pregnancy associated with cocaine use include abruptio placentae, preterm labor, precipitous birth, and stillbirth. Heroin is an opiate; its use in pregnancy is associated with preeclampsia, intrauterine growth restriction, miscarriage, premature rupture of membranes, infections, breech presentation, and preterm labor. The most serious effect of alcohol use in pregnancy is FAS. The major concern regarding PCP use in pregnant women is its association with polydrug abuse and its neurobehavioral effects on the neonate.

If nonsurgical treatment for late PPH is ineffective, which surgical procedure would be appropriate to correct the cause of this condition? a. Hysterectomy b. Laparoscopy c. Laparotomy d. Dilation and curettage (D&C)

ANS: D D&C allows the examination of the uterine contents and the removal of any retained placental fragments or blood clots. Hysterectomy is the removal of the uterus and is not the appropriate treatment for late PPH. A laparoscopy is the insertion of an endoscope through the abdominal wall to examine the peritoneal cavity, but it, too, is not the appropriate treatment for this condition. A laparotomy is the surgical incision into the peritoneal cavity to explore it but is also not the appropriate treatment for late PPH.

The nurse has evaluated a client with preeclampsia by assessing DTRs. The result is a grade of 3+. Which DTR response most accurately describes this score? a. Sluggish or diminished b. Brisk, hyperactive, with intermittent or transient clonus c. Active or expected response d. More brisk than expected, slightly hyperactive

ANS: D DTRs reflect the balance between the cerebral cortex and the spinal cord. They are evaluated at baseline and to detect changes. A slightly hyperactive and brisk response indicates a grade 3+ response.

Nurses need to understand the basic definitions and incidence data regarding PPH. Which statement regarding this condition is most accurate? a. PPH is easy to recognize early; after all, the woman is bleeding. b. Traditionally, it takes more than 1000 ml of blood after vaginal birth and 2500 ml after cesarean birth to define the condition as PPH. c. If anything, nurses and physicians tend to overestimate the amount of blood loss. d. Traditionally, PPH has been classified as early PPH or late PPH with respect to birth.

ANS: D Early PPH is also known as primary, or acute, PPH; late PPH is known as secondary PPH. Unfortunately, PPH can occur with little warning and is often recognized only after the mother has profound symptoms. Traditionally, a 500-ml blood loss after a vaginal birth and a 1000-ml blood loss after a cesarean birth constitute PPH. Medical personnel tend to underestimate blood loss by as much as 50% in their subjective observations.

The nurse is performing an assessment on a client who thinks she may be experiencing preterm labor. Which information is the most important for the nurse to understand and share with the client? a. Because all women must be considered at risk for preterm labor and prediction is so variable, teaching pregnant women the symptoms of preterm labor probably causes more harm through false alarms. b. Braxton Hicks contractions often signal the onset of preterm labor. c. Because preterm labor is likely to be the start of an extended labor, a woman with symptoms can wait several hours before contacting the primary caregiver. d. Diagnosis of preterm labor is based on gestational age, uterine activity, and progressive cervical change.

ANS: D Gestational age of 20 to 37 weeks, uterine contractions, and a cervix that is 80% effaced or dilated 2 cm indicates preterm labor. It is essential that nurses teach women how to detect the early symptoms of preterm labor. Braxton Hicks contractions resemble preterm labor contractions, but they are not true labor. Waiting too long to see a health care provider could result in essential medications failing to be administered. Preterm labor is not necessarily long-term labor.

The nurse suspects that her postpartum client is experiencing hemorrhagic shock. Which observation indicates or would confirm this diagnosis? a. Absence of cyanosis in the buccal mucosa b. Cool, dry skin c. Calm mental status d. Urinary output of at least 30 ml/hr

ANS: D Hemorrhage may result in hemorrhagic shock. Shock is an emergency situation during which the perfusion of body organs may become severely compromised, and death may occur. The presence of adequate urinary output indicates adequate tissue perfusion. The assessment of the buccal mucosa for cyanosis can be subjective. The presence of cool, pale, clammy skin is associated with hemorrhagic shock. Hemorrhagic shock is associated with lethargy, not restlessness.

The client has been on magnesium sulfate for 20 hours for the treatment of preeclampsia. She just delivered a viable infant girl 30 minutes ago. What uterine findings does the nurse expect to observe or assess in this client? a. Absence of uterine bleeding in the postpartum period b. Fundus firm below the level of the umbilicus c. Scant lochia flow d. Boggy uterus with heavy lochia flow

ANS: D High serum levels of magnesium can cause a relaxation of smooth muscle such as the uterus. Because of this tocolytic effect, the client will most likely have a boggy uterus with increased amounts of bleeding. All women experience uterine bleeding in the postpartum period, especially those who have received magnesium therapy. Rather than scant lochial flow, however, this client will most likely have a heavy flow attributable to the relaxation of the uterine wall caused by magnesium administration.

A woman with preeclampsia has a seizure. What is the nurses highest priority during a seizure? a. To insert an oral airway b. To suction the mouth to prevent aspiration c. To administer oxygen by mask d. To stay with the client and call for help

ANS: D If a client becomes eclamptic, then the nurse should stay with the client and call for help. Nursing actions during a convulsion are directed toward ensuring a patent airway and client safety. Insertion of an oral airway during seizure activity is no longer the standard of care. The nurse should attempt to keep the airway patent by turning the clients head to the side to prevent aspiration. Once the seizure has ended, it may be necessary to suction the clients mouth. Oxygen is administered after the convulsion has ended.

Which statement related to the induction of labor is most accurate? a. Can be achieved by external and internal version techniques b. Is also known as a trial of labor (TOL) c. Is almost always performed for medical reasons d. Is rated for viability by a Bishop score

ANS: D Induction of labor is likely to be more successful with a Bishop score of 9 or higher for first-time mothers or 5 or higher for veterans. Version is the turning of the fetus to a better position by a physician for an easier or safer birth. A TOL is the observance of a woman and her fetus for several hours of active labor to assess the safety of vaginal birth. Two thirds of cases of induced labor are elective and not done for medical reasons.

22. When would an internal version be indicated to manipulate the fetus into a vertex position? a. Fetus from a breech to a cephalic presentation before labor begins b. Fetus from a transverse lie to a longitudinal lie before a cesarean birth c. Second twin from an oblique lie to a transverse lie before labor begins d. Second twin from a transverse lie to a breech presentation during a vaginal birth

ANS: D Internal version is used only during a vaginal birth to manipulate the second twin into a presentation that allows it to be vaginally born. For internal version to occur, the cervix needs to be completely dilated.

Which classification of placental separation is not recognized as an abnormal adherence pattern? a. Placenta accreta b. Placenta increta c. Placenta percreta d. Placenta abruptio

ANS: D Placenta abruptio is premature separation of the placenta as opposed to partial or complete adherence. This classification occurs between the 20th week of gestation and delivery in the area of the decidua basalis. Symptoms include localized pain and bleeding. Placenta accreta is a recognized degree of attachment. With placenta accreta, the trophoblast slightly penetrates into the myometrium. Placenta increta is a recognized degree of attachment that results in deep penetration of the myometrium. Placenta percreta is the most severe degree of placental penetration that results in deep penetration of the myometrium. Bleeding with complete placental attachment occurs only when separation of the placenta is attempted after delivery. Treatment includes blood component therapy and, in extreme cases, hysterectomy may be necessary.

Which physiologic alteration of pregnancy most significantly affects glucose metabolism? a. Pancreatic function in the islets of Langerhans is affected by pregnancy. b. Pregnant women use glucose at a more rapid rate than nonpregnant women. c. Pregnant women significantly increase their dietary intake. d. Placental hormones are antagonistic to insulin, thus resulting in insulin resistance.

ANS: D Placental hormones, estrogen, progesterone, and human placental lactogen (HPL) create insulin resistance. Insulin is also broken down more quickly by the enzyme placental insulinase. Pancreatic functioning is not affected by pregnancy. The glucose requirements differ because of the growing fetus. The pregnant woman should increase her intake by 200 calories a day.

What is one of the initial signs and symptoms of puerperal infection in the postpartum client? a. Fatigue continuing for longer than 1 week b. Pain with voiding c. Profuse vaginal lochia with ambulation d. Temperature of 38 C (100.4 F) or higher on 2 successive days

ANS: D Postpartum or puerperal infection is any clinical infection of the genital canal that occurs within 28 days after miscarriage, induced abortion, or childbirth. The definition used in the United States continues to be the presence of a fever of 38 C (100.4 F) or higher on 2 successive days of the first 10 postpartum days, starting 24 hours after birth. Fatigue is a late finding associated with infection. Pain with voiding may indicate a urinary tract infection (UTI), but it is not typically one of the earlier symptoms of infection. Profuse lochia may be associated with endometritis, but it is not the first symptom associated with infection.

A 32-year-old primigravida is admitted with a diagnosis of ectopic pregnancy. Which information assists the nurse in developing the plan of care? a. Bed rest and analgesics are the recommended treatment. b. She will be unable to conceive in the future. c. A D&C will be performed to remove the products of conception. d. Hemorrhage is the primary concern.

ANS: D Severe bleeding occurs if the fallopian tube ruptures. The recommended treatment is to remove the pregnancy before rupture to prevent hemorrhaging. If the tube must be removed, then the womans fertility will decrease; however, she will not be infertile. A D&C is performed on the inside of the uterine cavity. The ectopic pregnancy is located within the tubes.

As part of the discharge teaching, the nurse can prepare the mother for her upcoming adjustment to her new role by instructing her regarding self-care activities to help prevent PPD. Which statement regarding this condition ismost helpful for the client? a. Stay home, and avoid outside activities to ensure adequate rest. b. Be certain that you are the only caregiver for your baby to facilitate infant attachment. c. Keep your feelings of sadness and adjustment to your new role to yourself. d. Realize that PPD is a common occurrence that affects many women.

ANS: D Should the new mother experience symptoms of the baby blues, it is important that she be aware that these symptoms are nothing to be ashamed of. As many as 10% to 15% of new mothers experience similar symptoms. Although obtaining enough rest is important for the mother, she should not distance herself from her family and friends. Her spouse or partner can communicate the best visiting times to enable the new mother to obtain adequate rest. It is also important that she not isolate herself at home by herself during this time of role adjustment. Even if breastfeeding, other family members can participate in the infants care. If depression occurs, then the symptoms will often interfere with mothering functions; therefore, family support is essential. The new mother should share her feelings with someone else and avoid overcommitting herself or feel as though she has to besuperwoman. A telephone call to the hospital warm line may provide reassurance with lactation issues and other infant care questions. Should symptoms continue, a referral to a professional therapist may be necessary

The obstetric provider has informed the nurse that she will be performing an amniotomy on the client to induce labor. What is the nurses highest priority intervention after the amniotomy is performed? a. Applying clean linens under the woman b. Taking the clients vital signs c. Performing a vaginal examination d. Assessing the fetal heart rate (FHR)

ANS: D The FHR is assessed before and immediately after the amniotomy to detect any changes that might indicate cord compression or prolapse. Providing comfort measures, such as clean linens, for the client is important but not the priority immediately after an amniotomy. The womans temperature should be checked every 2 hours after the rupture of membranes but not the priority immediately after an amniotomy. The woman would have had a vaginal examination during the procedure. Unless cord prolapse is suspected, another vaginal examination is not warranted. Additionally, FHR assessment provides clinical cues to a prolapsed cord.

What is the most dangerous effect on the fetus of a mother who smokes cigarettes while pregnant? a. Genetic changes and anomalies b. Extensive CNS damage c. Fetal addiction to the substance inhaled d. Intrauterine growth restriction

ANS: D The major consequences of smoking tobacco during pregnancy are low-birth-weight infants, prematurity, and increased perinatal loss. Cigarettes will not normally cause genetic changes or extensive CNS damage. Addiction to tobacco is not a usual concern related to the neonate.

In evaluating the effectiveness of magnesium sulfate for the treatment of preterm labor, which finding alerts the nurse to possible side effects? a. Urine output of 160 ml in 4 hours b. DTRs 2+ and no clonus c. Respiratory rate (RR) of 16 breaths per minute d. Serum magnesium level of 10 mg/dl

ANS: D The therapeutic range for magnesium sulfate management is 4 to 7.5 mg/dl. A serum magnesium level of 10 mg/dl could lead to signs and symptoms of magnesium toxicity, including oliguria and respiratory distress. Urine output of 160 ml in 4 hours, DTRs of 2+, and a RR of 16 breaths per minute are all normal findings.

A woman at 39 weeks of gestation with a history of preeclampsia is admitted to the labor and birth unit. She suddenly experiences increased contraction frequency of every 1 to 2 minutes, dark red vaginal bleeding, and a tense, painful abdomen. Which clinical change does the nurse anticipate? a. Eclamptic seizure b. Rupture of the uterus c. Placenta previa d. Abruptio placentae

ANS: D Uterine tenderness in the presence of increasing tone may be the earliest sign of abruptio placentae. Women with preeclampsia are at increased risk for an abruption attributable to decreased placental perfusion. Eclamptic seizures are evidenced by the presence of generalized tonic-clonic convulsions. Uterine rupture exhibits hypotonic uterine activity, signs of hypovolemia, and, in many cases, the absence of pain. Placenta previa exhibits bright red, painless vaginal bleeding.

Which statement best describes chronic hypertension? a. Chronic hypertension is defined as hypertension that begins during pregnancy and lasts for the duration of the pregnancy. b. Chronic hypertension is considered severe when the systolic BP is higher than 140 mm Hg or the diastolic BP is higher than 90 mm Hg. c. Chronic hypertension is general hypertension plus proteinuria. d. Chronic hypertension can occur independently of or simultaneously with preeclampsia.

ANS: D Women with chronic hypertension may develop superimposed preeclampsia, which increases the morbidity for both the mother and the fetus. Chronic hypertension is present before pregnancy or diagnosed before the 20 weeks of gestation and persists longer than 6 weeks postpartum. Chronic hypertension becomes severe with a diastolic BP of 110 mm Hg or higher. Proteinuria is an excessive concentration of protein in the urine and is a complication of hypertension, not a defining characteristic.


Conjuntos de estudio relacionados

Managing Ppl/Org - Values and Attitudes

View Set

cross sectional male pelvis, scrotal & penile sonography

View Set

Gazdasagi szereplők, piacok, kereslet, kínálat

View Set

Tinker v. Des Moines Independent Community School District (1969)

View Set

Research Methods Exam 1- Chapter 2: Sources of Info

View Set